Difference between revisions of "2002 AMC 10B Problems/Problem 25"

m (Problem)
Line 6: Line 6:
 
== Solution ==
 
== Solution ==
 
Let <math>x</math> be the sum of the integers and <math>y</math> be the number of elements in the list. Then we get the equations <math>\frac{x+15}{y+1}=\frac{x}{y}+2</math> and <math>\frac{x+15+1}{y+1+1}=\frac{x+16}{y+2}=\frac{x}{y}+2-1=\frac{x}{y}+1</math>. With a little work, the solution is found to be <math>y= \boxed{\textbf{(A)}\ 4} </math>.
 
Let <math>x</math> be the sum of the integers and <math>y</math> be the number of elements in the list. Then we get the equations <math>\frac{x+15}{y+1}=\frac{x}{y}+2</math> and <math>\frac{x+15+1}{y+1+1}=\frac{x+16}{y+2}=\frac{x}{y}+2-1=\frac{x}{y}+1</math>. With a little work, the solution is found to be <math>y= \boxed{\textbf{(A)}\ 4} </math>.
 +
 +
== See also ==
 +
{{AMC10 box|year=2002|ab=B|num-b=24|num-a=Last Problem}}

Revision as of 19:28, 28 December 2011

Problem

When $15$ is appended to a list of integers, the mean is increased by $2$. When $1$ is appended to the enlarged list, the mean of the enlarged list is decreased by $1$. How many integers were in the original list?

$\mathrm{(A) \ } 4\qquad \mathrm{(B) \ } 5\qquad \mathrm{(C) \ } 6\qquad \mathrm{(D) \ } 7\qquad \mathrm{(E) \ } 8$

Solution

Let $x$ be the sum of the integers and $y$ be the number of elements in the list. Then we get the equations $\frac{x+15}{y+1}=\frac{x}{y}+2$ and $\frac{x+15+1}{y+1+1}=\frac{x+16}{y+2}=\frac{x}{y}+2-1=\frac{x}{y}+1$. With a little work, the solution is found to be $y= \boxed{\textbf{(A)}\ 4}$.

See also

2002 AMC 10B (ProblemsAnswer KeyResources)
Preceded by
Problem 24
Followed by
Problem Last Problem
1 2 3 4 5 6 7 8 9 10 11 12 13 14 15 16 17 18 19 20 21 22 23 24 25
All AMC 10 Problems and Solutions